I need help with a competition math problem please. [duplicate]












0












$begingroup$



This question already has an answer here:




  • Solving a polynomial with grouping

    1 answer




I am a competition math student, and I was doing a practice test when I found this problem:
"The fourth degree polynomial equation $x^4 - 7x^3 + 4x^2 + 7x - 4 = 0$ has four real roots a, b, c and d. What is the value of the sum $1/a + 1/b + 1/c + 1/d$? Express your answer as a common fraction."
First of all, I don't think this polynomial can be factored, so I don't know how to find the reciprocals of the roots. I need help and any help would be appreciated! Note: When they say "Express your answer as a common fraction.", it means that the answer IS a fraction. I tried asking this question already, but I worded it differently, and they didn't get a fraction so I knew it was wrong. This is the link to my other question: Link










share|cite|improve this question











$endgroup$



marked as duplicate by jmerry, Andrew, Robert Soupe, Lord Shark the Unknown, Leucippus Jan 30 at 8:15


This question has been asked before and already has an answer. If those answers do not fully address your question, please ask a new question.


















  • $begingroup$
    You may find Vieta's formulas useful.
    $endgroup$
    – nathan.j.mcdougall
    Jan 30 at 0:23










  • $begingroup$
    Make the substitution $y=1/x$ and consider the quartic in $y$.
    $endgroup$
    – Donald Splutterwit
    Jan 30 at 0:23










  • $begingroup$
    See here
    $endgroup$
    – J. W. Tanner
    Jan 30 at 0:34










  • $begingroup$
    @J.W.Tanner It isn't a duplicate. Read my edit.
    $endgroup$
    – Spencer1O1
    Jan 30 at 0:42










  • $begingroup$
    The answer there has been corrected now
    $endgroup$
    – J. W. Tanner
    Jan 30 at 17:40
















0












$begingroup$



This question already has an answer here:




  • Solving a polynomial with grouping

    1 answer




I am a competition math student, and I was doing a practice test when I found this problem:
"The fourth degree polynomial equation $x^4 - 7x^3 + 4x^2 + 7x - 4 = 0$ has four real roots a, b, c and d. What is the value of the sum $1/a + 1/b + 1/c + 1/d$? Express your answer as a common fraction."
First of all, I don't think this polynomial can be factored, so I don't know how to find the reciprocals of the roots. I need help and any help would be appreciated! Note: When they say "Express your answer as a common fraction.", it means that the answer IS a fraction. I tried asking this question already, but I worded it differently, and they didn't get a fraction so I knew it was wrong. This is the link to my other question: Link










share|cite|improve this question











$endgroup$



marked as duplicate by jmerry, Andrew, Robert Soupe, Lord Shark the Unknown, Leucippus Jan 30 at 8:15


This question has been asked before and already has an answer. If those answers do not fully address your question, please ask a new question.


















  • $begingroup$
    You may find Vieta's formulas useful.
    $endgroup$
    – nathan.j.mcdougall
    Jan 30 at 0:23










  • $begingroup$
    Make the substitution $y=1/x$ and consider the quartic in $y$.
    $endgroup$
    – Donald Splutterwit
    Jan 30 at 0:23










  • $begingroup$
    See here
    $endgroup$
    – J. W. Tanner
    Jan 30 at 0:34










  • $begingroup$
    @J.W.Tanner It isn't a duplicate. Read my edit.
    $endgroup$
    – Spencer1O1
    Jan 30 at 0:42










  • $begingroup$
    The answer there has been corrected now
    $endgroup$
    – J. W. Tanner
    Jan 30 at 17:40














0












0








0


2



$begingroup$



This question already has an answer here:




  • Solving a polynomial with grouping

    1 answer




I am a competition math student, and I was doing a practice test when I found this problem:
"The fourth degree polynomial equation $x^4 - 7x^3 + 4x^2 + 7x - 4 = 0$ has four real roots a, b, c and d. What is the value of the sum $1/a + 1/b + 1/c + 1/d$? Express your answer as a common fraction."
First of all, I don't think this polynomial can be factored, so I don't know how to find the reciprocals of the roots. I need help and any help would be appreciated! Note: When they say "Express your answer as a common fraction.", it means that the answer IS a fraction. I tried asking this question already, but I worded it differently, and they didn't get a fraction so I knew it was wrong. This is the link to my other question: Link










share|cite|improve this question











$endgroup$





This question already has an answer here:




  • Solving a polynomial with grouping

    1 answer




I am a competition math student, and I was doing a practice test when I found this problem:
"The fourth degree polynomial equation $x^4 - 7x^3 + 4x^2 + 7x - 4 = 0$ has four real roots a, b, c and d. What is the value of the sum $1/a + 1/b + 1/c + 1/d$? Express your answer as a common fraction."
First of all, I don't think this polynomial can be factored, so I don't know how to find the reciprocals of the roots. I need help and any help would be appreciated! Note: When they say "Express your answer as a common fraction.", it means that the answer IS a fraction. I tried asking this question already, but I worded it differently, and they didn't get a fraction so I knew it was wrong. This is the link to my other question: Link





This question already has an answer here:




  • Solving a polynomial with grouping

    1 answer








algebra-precalculus polynomials






share|cite|improve this question















share|cite|improve this question













share|cite|improve this question




share|cite|improve this question








edited Jan 30 at 0:40







Spencer1O1

















asked Jan 30 at 0:19









Spencer1O1Spencer1O1

125




125




marked as duplicate by jmerry, Andrew, Robert Soupe, Lord Shark the Unknown, Leucippus Jan 30 at 8:15


This question has been asked before and already has an answer. If those answers do not fully address your question, please ask a new question.









marked as duplicate by jmerry, Andrew, Robert Soupe, Lord Shark the Unknown, Leucippus Jan 30 at 8:15


This question has been asked before and already has an answer. If those answers do not fully address your question, please ask a new question.














  • $begingroup$
    You may find Vieta's formulas useful.
    $endgroup$
    – nathan.j.mcdougall
    Jan 30 at 0:23










  • $begingroup$
    Make the substitution $y=1/x$ and consider the quartic in $y$.
    $endgroup$
    – Donald Splutterwit
    Jan 30 at 0:23










  • $begingroup$
    See here
    $endgroup$
    – J. W. Tanner
    Jan 30 at 0:34










  • $begingroup$
    @J.W.Tanner It isn't a duplicate. Read my edit.
    $endgroup$
    – Spencer1O1
    Jan 30 at 0:42










  • $begingroup$
    The answer there has been corrected now
    $endgroup$
    – J. W. Tanner
    Jan 30 at 17:40


















  • $begingroup$
    You may find Vieta's formulas useful.
    $endgroup$
    – nathan.j.mcdougall
    Jan 30 at 0:23










  • $begingroup$
    Make the substitution $y=1/x$ and consider the quartic in $y$.
    $endgroup$
    – Donald Splutterwit
    Jan 30 at 0:23










  • $begingroup$
    See here
    $endgroup$
    – J. W. Tanner
    Jan 30 at 0:34










  • $begingroup$
    @J.W.Tanner It isn't a duplicate. Read my edit.
    $endgroup$
    – Spencer1O1
    Jan 30 at 0:42










  • $begingroup$
    The answer there has been corrected now
    $endgroup$
    – J. W. Tanner
    Jan 30 at 17:40
















$begingroup$
You may find Vieta's formulas useful.
$endgroup$
– nathan.j.mcdougall
Jan 30 at 0:23




$begingroup$
You may find Vieta's formulas useful.
$endgroup$
– nathan.j.mcdougall
Jan 30 at 0:23












$begingroup$
Make the substitution $y=1/x$ and consider the quartic in $y$.
$endgroup$
– Donald Splutterwit
Jan 30 at 0:23




$begingroup$
Make the substitution $y=1/x$ and consider the quartic in $y$.
$endgroup$
– Donald Splutterwit
Jan 30 at 0:23












$begingroup$
See here
$endgroup$
– J. W. Tanner
Jan 30 at 0:34




$begingroup$
See here
$endgroup$
– J. W. Tanner
Jan 30 at 0:34












$begingroup$
@J.W.Tanner It isn't a duplicate. Read my edit.
$endgroup$
– Spencer1O1
Jan 30 at 0:42




$begingroup$
@J.W.Tanner It isn't a duplicate. Read my edit.
$endgroup$
– Spencer1O1
Jan 30 at 0:42












$begingroup$
The answer there has been corrected now
$endgroup$
– J. W. Tanner
Jan 30 at 17:40




$begingroup$
The answer there has been corrected now
$endgroup$
– J. W. Tanner
Jan 30 at 17:40










2 Answers
2






active

oldest

votes


















0












$begingroup$

Suppose we have a polynomial $f(x)=a_nx^n+a_{n-1}x^{n-1}+dots +a_0$ with roots $r_1,r_2,dots, r_n$. We claim that the polynomial with roots $frac{1}{r_1},frac{1}{r_2},dots,frac{1}{r_n}$ is simply $g(x)=a_0x^n+a_1x^{n-1}+dots + a_n$. Why? Because if you observe that the roots of the polynomial $f(frac{1}{x})$ are the reciprocals of the roots of $f(x)$, then our desired polynomial is $$begin{align*}g(x)&=x^nf(1/x)\&=x^nleft(frac{a_n}{x^n}+frac{a_{n-1}} {x^{n-1}}+cdots+frac{a_1}{x}+a_0right)\&=a_0x^n+a_1x^{n-1}+cdots+a_n,end{align*}$$



The desired polynomial can be found by simply 'reversing the coefficients' as shown above. Hence the polynomial with roots $frac{1}{a},frac 1b, frac 1c,$ and $frac 1d$ is $$-4x^4+7x^3+4x^2-7x+1$$ Applying Vieta's Formulae to this, we find that the sum of the roots is $-frac{-7}{4}=frac{7}{4}$.






share|cite|improve this answer











$endgroup$









  • 1




    $begingroup$
    Thanks! I got 7/4 too because my reasoning was that $x^4 p(1/x) = -4x^4+7x^3+4x^2-7x+1$ and the sum of the roots is $-7/-4=7/4$. I just wasn't 100% sure that you could do this.
    $endgroup$
    – Spencer1O1
    Jan 30 at 0:38










  • $begingroup$
    @Spencer1O1 nice work! In the future, consider tagging your competition math questions with the tag [contest-math]
    $endgroup$
    – user574848
    Jan 30 at 0:40










  • $begingroup$
    Okay! I tried to tag it with [competition-math], but it didn't exist. Thanks for the tip!
    $endgroup$
    – Spencer1O1
    Jan 30 at 0:46



















3












$begingroup$

The trick is to write
$$
frac{1}{a}+
frac{1}{b}+
frac{1}{c}+
frac{1}{d}=frac{bcd+acd+abd+abc}{abcd}
$$

write $f(x)=x^4-7x^3+4x^2+7x-4=(x-a)(x-b)(x-c)(x-d)$ and use Vieta's relations.



For example, by multiplying out we see at once that $abcd=-4$ and $-(bcd+acd+abd+abc)=7.$






share|cite|improve this answer









$endgroup$




















    2 Answers
    2






    active

    oldest

    votes








    2 Answers
    2






    active

    oldest

    votes









    active

    oldest

    votes






    active

    oldest

    votes









    0












    $begingroup$

    Suppose we have a polynomial $f(x)=a_nx^n+a_{n-1}x^{n-1}+dots +a_0$ with roots $r_1,r_2,dots, r_n$. We claim that the polynomial with roots $frac{1}{r_1},frac{1}{r_2},dots,frac{1}{r_n}$ is simply $g(x)=a_0x^n+a_1x^{n-1}+dots + a_n$. Why? Because if you observe that the roots of the polynomial $f(frac{1}{x})$ are the reciprocals of the roots of $f(x)$, then our desired polynomial is $$begin{align*}g(x)&=x^nf(1/x)\&=x^nleft(frac{a_n}{x^n}+frac{a_{n-1}} {x^{n-1}}+cdots+frac{a_1}{x}+a_0right)\&=a_0x^n+a_1x^{n-1}+cdots+a_n,end{align*}$$



    The desired polynomial can be found by simply 'reversing the coefficients' as shown above. Hence the polynomial with roots $frac{1}{a},frac 1b, frac 1c,$ and $frac 1d$ is $$-4x^4+7x^3+4x^2-7x+1$$ Applying Vieta's Formulae to this, we find that the sum of the roots is $-frac{-7}{4}=frac{7}{4}$.






    share|cite|improve this answer











    $endgroup$









    • 1




      $begingroup$
      Thanks! I got 7/4 too because my reasoning was that $x^4 p(1/x) = -4x^4+7x^3+4x^2-7x+1$ and the sum of the roots is $-7/-4=7/4$. I just wasn't 100% sure that you could do this.
      $endgroup$
      – Spencer1O1
      Jan 30 at 0:38










    • $begingroup$
      @Spencer1O1 nice work! In the future, consider tagging your competition math questions with the tag [contest-math]
      $endgroup$
      – user574848
      Jan 30 at 0:40










    • $begingroup$
      Okay! I tried to tag it with [competition-math], but it didn't exist. Thanks for the tip!
      $endgroup$
      – Spencer1O1
      Jan 30 at 0:46
















    0












    $begingroup$

    Suppose we have a polynomial $f(x)=a_nx^n+a_{n-1}x^{n-1}+dots +a_0$ with roots $r_1,r_2,dots, r_n$. We claim that the polynomial with roots $frac{1}{r_1},frac{1}{r_2},dots,frac{1}{r_n}$ is simply $g(x)=a_0x^n+a_1x^{n-1}+dots + a_n$. Why? Because if you observe that the roots of the polynomial $f(frac{1}{x})$ are the reciprocals of the roots of $f(x)$, then our desired polynomial is $$begin{align*}g(x)&=x^nf(1/x)\&=x^nleft(frac{a_n}{x^n}+frac{a_{n-1}} {x^{n-1}}+cdots+frac{a_1}{x}+a_0right)\&=a_0x^n+a_1x^{n-1}+cdots+a_n,end{align*}$$



    The desired polynomial can be found by simply 'reversing the coefficients' as shown above. Hence the polynomial with roots $frac{1}{a},frac 1b, frac 1c,$ and $frac 1d$ is $$-4x^4+7x^3+4x^2-7x+1$$ Applying Vieta's Formulae to this, we find that the sum of the roots is $-frac{-7}{4}=frac{7}{4}$.






    share|cite|improve this answer











    $endgroup$









    • 1




      $begingroup$
      Thanks! I got 7/4 too because my reasoning was that $x^4 p(1/x) = -4x^4+7x^3+4x^2-7x+1$ and the sum of the roots is $-7/-4=7/4$. I just wasn't 100% sure that you could do this.
      $endgroup$
      – Spencer1O1
      Jan 30 at 0:38










    • $begingroup$
      @Spencer1O1 nice work! In the future, consider tagging your competition math questions with the tag [contest-math]
      $endgroup$
      – user574848
      Jan 30 at 0:40










    • $begingroup$
      Okay! I tried to tag it with [competition-math], but it didn't exist. Thanks for the tip!
      $endgroup$
      – Spencer1O1
      Jan 30 at 0:46














    0












    0








    0





    $begingroup$

    Suppose we have a polynomial $f(x)=a_nx^n+a_{n-1}x^{n-1}+dots +a_0$ with roots $r_1,r_2,dots, r_n$. We claim that the polynomial with roots $frac{1}{r_1},frac{1}{r_2},dots,frac{1}{r_n}$ is simply $g(x)=a_0x^n+a_1x^{n-1}+dots + a_n$. Why? Because if you observe that the roots of the polynomial $f(frac{1}{x})$ are the reciprocals of the roots of $f(x)$, then our desired polynomial is $$begin{align*}g(x)&=x^nf(1/x)\&=x^nleft(frac{a_n}{x^n}+frac{a_{n-1}} {x^{n-1}}+cdots+frac{a_1}{x}+a_0right)\&=a_0x^n+a_1x^{n-1}+cdots+a_n,end{align*}$$



    The desired polynomial can be found by simply 'reversing the coefficients' as shown above. Hence the polynomial with roots $frac{1}{a},frac 1b, frac 1c,$ and $frac 1d$ is $$-4x^4+7x^3+4x^2-7x+1$$ Applying Vieta's Formulae to this, we find that the sum of the roots is $-frac{-7}{4}=frac{7}{4}$.






    share|cite|improve this answer











    $endgroup$



    Suppose we have a polynomial $f(x)=a_nx^n+a_{n-1}x^{n-1}+dots +a_0$ with roots $r_1,r_2,dots, r_n$. We claim that the polynomial with roots $frac{1}{r_1},frac{1}{r_2},dots,frac{1}{r_n}$ is simply $g(x)=a_0x^n+a_1x^{n-1}+dots + a_n$. Why? Because if you observe that the roots of the polynomial $f(frac{1}{x})$ are the reciprocals of the roots of $f(x)$, then our desired polynomial is $$begin{align*}g(x)&=x^nf(1/x)\&=x^nleft(frac{a_n}{x^n}+frac{a_{n-1}} {x^{n-1}}+cdots+frac{a_1}{x}+a_0right)\&=a_0x^n+a_1x^{n-1}+cdots+a_n,end{align*}$$



    The desired polynomial can be found by simply 'reversing the coefficients' as shown above. Hence the polynomial with roots $frac{1}{a},frac 1b, frac 1c,$ and $frac 1d$ is $$-4x^4+7x^3+4x^2-7x+1$$ Applying Vieta's Formulae to this, we find that the sum of the roots is $-frac{-7}{4}=frac{7}{4}$.







    share|cite|improve this answer














    share|cite|improve this answer



    share|cite|improve this answer








    edited Jan 30 at 0:38

























    answered Jan 30 at 0:30









    user574848user574848

    688118




    688118








    • 1




      $begingroup$
      Thanks! I got 7/4 too because my reasoning was that $x^4 p(1/x) = -4x^4+7x^3+4x^2-7x+1$ and the sum of the roots is $-7/-4=7/4$. I just wasn't 100% sure that you could do this.
      $endgroup$
      – Spencer1O1
      Jan 30 at 0:38










    • $begingroup$
      @Spencer1O1 nice work! In the future, consider tagging your competition math questions with the tag [contest-math]
      $endgroup$
      – user574848
      Jan 30 at 0:40










    • $begingroup$
      Okay! I tried to tag it with [competition-math], but it didn't exist. Thanks for the tip!
      $endgroup$
      – Spencer1O1
      Jan 30 at 0:46














    • 1




      $begingroup$
      Thanks! I got 7/4 too because my reasoning was that $x^4 p(1/x) = -4x^4+7x^3+4x^2-7x+1$ and the sum of the roots is $-7/-4=7/4$. I just wasn't 100% sure that you could do this.
      $endgroup$
      – Spencer1O1
      Jan 30 at 0:38










    • $begingroup$
      @Spencer1O1 nice work! In the future, consider tagging your competition math questions with the tag [contest-math]
      $endgroup$
      – user574848
      Jan 30 at 0:40










    • $begingroup$
      Okay! I tried to tag it with [competition-math], but it didn't exist. Thanks for the tip!
      $endgroup$
      – Spencer1O1
      Jan 30 at 0:46








    1




    1




    $begingroup$
    Thanks! I got 7/4 too because my reasoning was that $x^4 p(1/x) = -4x^4+7x^3+4x^2-7x+1$ and the sum of the roots is $-7/-4=7/4$. I just wasn't 100% sure that you could do this.
    $endgroup$
    – Spencer1O1
    Jan 30 at 0:38




    $begingroup$
    Thanks! I got 7/4 too because my reasoning was that $x^4 p(1/x) = -4x^4+7x^3+4x^2-7x+1$ and the sum of the roots is $-7/-4=7/4$. I just wasn't 100% sure that you could do this.
    $endgroup$
    – Spencer1O1
    Jan 30 at 0:38












    $begingroup$
    @Spencer1O1 nice work! In the future, consider tagging your competition math questions with the tag [contest-math]
    $endgroup$
    – user574848
    Jan 30 at 0:40




    $begingroup$
    @Spencer1O1 nice work! In the future, consider tagging your competition math questions with the tag [contest-math]
    $endgroup$
    – user574848
    Jan 30 at 0:40












    $begingroup$
    Okay! I tried to tag it with [competition-math], but it didn't exist. Thanks for the tip!
    $endgroup$
    – Spencer1O1
    Jan 30 at 0:46




    $begingroup$
    Okay! I tried to tag it with [competition-math], but it didn't exist. Thanks for the tip!
    $endgroup$
    – Spencer1O1
    Jan 30 at 0:46











    3












    $begingroup$

    The trick is to write
    $$
    frac{1}{a}+
    frac{1}{b}+
    frac{1}{c}+
    frac{1}{d}=frac{bcd+acd+abd+abc}{abcd}
    $$

    write $f(x)=x^4-7x^3+4x^2+7x-4=(x-a)(x-b)(x-c)(x-d)$ and use Vieta's relations.



    For example, by multiplying out we see at once that $abcd=-4$ and $-(bcd+acd+abd+abc)=7.$






    share|cite|improve this answer









    $endgroup$


















      3












      $begingroup$

      The trick is to write
      $$
      frac{1}{a}+
      frac{1}{b}+
      frac{1}{c}+
      frac{1}{d}=frac{bcd+acd+abd+abc}{abcd}
      $$

      write $f(x)=x^4-7x^3+4x^2+7x-4=(x-a)(x-b)(x-c)(x-d)$ and use Vieta's relations.



      For example, by multiplying out we see at once that $abcd=-4$ and $-(bcd+acd+abd+abc)=7.$






      share|cite|improve this answer









      $endgroup$
















        3












        3








        3





        $begingroup$

        The trick is to write
        $$
        frac{1}{a}+
        frac{1}{b}+
        frac{1}{c}+
        frac{1}{d}=frac{bcd+acd+abd+abc}{abcd}
        $$

        write $f(x)=x^4-7x^3+4x^2+7x-4=(x-a)(x-b)(x-c)(x-d)$ and use Vieta's relations.



        For example, by multiplying out we see at once that $abcd=-4$ and $-(bcd+acd+abd+abc)=7.$






        share|cite|improve this answer









        $endgroup$



        The trick is to write
        $$
        frac{1}{a}+
        frac{1}{b}+
        frac{1}{c}+
        frac{1}{d}=frac{bcd+acd+abd+abc}{abcd}
        $$

        write $f(x)=x^4-7x^3+4x^2+7x-4=(x-a)(x-b)(x-c)(x-d)$ and use Vieta's relations.



        For example, by multiplying out we see at once that $abcd=-4$ and $-(bcd+acd+abd+abc)=7.$







        share|cite|improve this answer












        share|cite|improve this answer



        share|cite|improve this answer










        answered Jan 30 at 0:26









        Foobaz JohnFoobaz John

        22.9k41552




        22.9k41552















            Popular posts from this blog

            Can a sorcerer learn a 5th-level spell early by creating spell slots using the Font of Magic feature?

            Does disintegrating a polymorphed enemy still kill it after the 2018 errata?

            A Topological Invariant for $pi_3(U(n))$